respuesta de los siguientes derivados

Adjuntos:

Respuestas

Respuesta dada por: lionelayuso46528
1

Respuesta:

Tgzddtf6dd5t8yzydmufñYErs

Respuesta dada por: lopezsmith2004
1

Respuesta:

1. f (x) = 5x³

lim      = 5(x + h)³ - 5x³/h

h ⇒ 0

lim       = 5(x³ + 3x²h + 3xh² + h³) - 5x³/h

h ⇒ 0

lim       = 5x³ + 15x²h + 15xh² + h³ - 5x³/h

h ⇒ 0

lim       = 15x²h + 15xh² + h³/h

h ⇒ 0

lim       = h(15x² + 15xh + h²)/h

h ⇒ 0

lim       = 15x²

h ⇒ 0

Explicación paso a paso:


olga2002: gracias
Preguntas similares